difficulty understanding the question
I was confused why the question asked where (2others) can go if it was only 1 female left after p...
Elizabeth25 on March 25 at 11:12PM
  • June 2002 LSAT
  • SEC3
  • Q1
1
Reply
June2002-S3-Q24
The correct answer does not make sense to me because one of O's laps does not precede immediately...
maonuo on November 22 at 04:56PM
  • June 2002 LSAT
  • SEC3
  • Q24
1
Reply
Answer explanation
Can someone please explain all the right answers for all the questions
zia305 on November 22 at 04:54PM
  • June 2002 LSAT
  • SEC3
  • Q20
1
Reply
questions like these
For questions like these for example saying: if x and y are in places 1 and 2 for how many of the...
Abigail-Lee on November 22 at 04:51PM
  • June 2002 LSAT
  • SEC3
  • Q20
1
Reply
Why not D?
Wouldn't semantics have to be locked in on the first shelf?
Nic-Teodosescu on November 20 at 01:44PM
  • June 2002 LSAT
  • SEC3
  • Q18
1
Reply
determining color order
Hello, For this problem and the last one, I am having a difficult time determining color order....
Nic-Teodosescu on November 20 at 01:37PM
  • June 2002 LSAT
  • SEC3
  • Q8
1
Reply
Why is it not B?
Can you explain why B is incorrect?
Abigail-Lee on November 20 at 03:38AM
  • June 2002 LSAT
  • SEC3
  • Q5
1
Reply
Rule number 5
Why does the contrapositive not effect the North and South wing The explanation is Vs m -> Wn ...
Darian-Kashanikhah-2 on November 20 at 03:35AM
  • June 2002 LSAT
  • SEC3
  • Q1
1
Reply
Game Type
What kind of game type is this?
c0cald01 on November 20 at 03:32AM
  • June 2002 LSAT
  • SEC3
  • Q1
1
Reply
Q8 answer c
Why is the answer U and not S?
Samantha74 on April 27 at 06:21PM
  • June 2002 LSAT
  • SEC3
  • Q8
1
Reply
How can I improve on interpreting "determinatio...
Based on the previous post, I understand why I got this one wrong because of the way they phrased...
Angel92 on May 17, 2022
  • June 2002 LSAT
  • SEC3
  • Q10
2
Replies
Can someone please explain the answer to the qu...
I'm not sure how you figure out that the answer is two.
Kirsten on July 16, 2021
  • June 2002 LSAT
  • SEC3
  • Q10
3
Replies
Walk through
Can someone explain the question setup and walk through all the answers to this game. I do not u...
ddamico212 on November 2, 2020
  • June 2002 LSAT
  • SEC3
  • Q6
1
Reply
Game setup
Please help me with the dedications and set up on this one. Completely lost.
dallman0303 on October 28, 2020
  • June 2002 LSAT
  • SEC3
  • Q3
4
Replies
Swim team
So this is how I solved this ... Wondering if I am off? Set up and deductions: ~O ~K 1 6 ~J ...
Lily on September 12, 2020
  • June 2002 LSAT
  • SEC3
  • Q19
3
Replies
Video
The reply to the previous question, directs us to a video for explanation. I don't see it. If thi...
DDL on September 6, 2020
  • June 2002 LSAT
  • SEC3
  • Q19
4
Replies